I need to find the value of x?

I Need To Find The Value Of X?

Answers

Answer 1

Answer:

x=20

Step-by-step explanation:

120=6x

120/6=6x/6

20=x


Related Questions

Consider the expression 63+81 how can you use the distributive property and the gcf to find an equivalent expression?explain how you can check your answer

Answers

Step-by-step explanation:

63+81

gcf = 9

63÷9=7, 81÷9=9

so, 63+81 = (9×7)+(9×9)

= 9×(7+9)

63+81 = 144

9x(7+9) = 9×16 = 144

The top of the Boulder Dam has an angle of elevation of 1.2 radians from a point on the Colorado River. Measuring the angle of elevation to the top of the dam from a point 155 feet farther down river is 0.9 radi- ans; assume the two angle measurements are taken at the same elevation above sea level. How high is the dam?

Answers

Answer:

382.925 feets

Step-by-step explanation:

The solution diagram is attached below :

Converting radian measurement to degree :

radian angle * 180/π = degree angle

1.2 * 180/π = 68.755°

0.9 * 180/π = 51.566°

Height of dam is h:

Using trigonometry :

Tan θ = opposite / Adjacent

Tan 68.755° = h / x

h = x Tan 68.755° - - - (1)

Tan 51.566° = h / (155+x)

h = (155+x) tan 51.566° - - - (2)

Equate (1) and (2)

x Tan 68.755 = (155+x) Tan 51.566

x Tan 68.755 = 155tan 51.566 + x tan 51.566

x Tan 68.755 = 195.32311 + x Tan 51.566

x Tan 68.755 - x Tan 51.566 = 195.32311

x(tan 68.755 - tan 51.566) = 195.32311

x * 1.3120110 = 195.32311

1.3120110x = 195.32311

x = 195.32311 / 1.3120110

x = 148.87307

Using :

h = x Tan 68.755

h = 148.87307 * tan(68.755)

h = 382.92539

h = 382.925 feets

In a random sample of 150 customers of a high-speed internet provider, 63 said that their service had been interrupted one or more times in the past month. Find a 95% confidence interval for the proportion of customers whose service was interrupted one or more times in the past month.

Answers

Answer:

The correct answer is "0.3410, 0.4990".

Step-by-step explanation:

Given values are:

[tex]n=150[/tex]

[tex]p=\frac{63}{150}[/tex]

  [tex]=0.42[/tex]

At 95% confidence interval,

C = 95%

z = 1.96

As we know,

⇒ [tex]E=z\sqrt{\frac{p(1-p)}{n} }[/tex]

By substituting the values, we get

       [tex]=1.96\sqrt{\frac{0.42\times 0.58}{150} }[/tex]

       [tex]=1.96\sqrt{\frac{0.2436}{150} }[/tex]

       [tex]=0.0790[/tex]

hence,

The confidence interval will be:

= [tex]p \pm E[/tex]

= [tex]0.42 \pm 0.079[/tex]

= [tex](0.3410,0.4990)[/tex]

The probability distribution for a random variable x is given in the table X: -5,-3,-2,0,2,3 Probability: .17,.13,.33,.16,.11,.10 Find the probability that X <_-3

Answers

Answer:

0.3 probability that [tex]x \leq -3[/tex]

Step-by-step explanation:

The probability distribution is given in the table.

Probability that x <= -3

The values that are -3 or lower are -3 and -5. So

[tex]P(X \leq -3) = P(X = -3) + P(X = -5)[/tex]

From the table:

[tex]P(X = -3) = 0.13, P(X = -5) = 0.17[/tex]. So

[tex]P(X \leq -3) = P(X = -3) + P(X = -5) = 0.13 + 0.17 = 0.3[/tex]

0.3 probability that [tex]x \leq -3[/tex]

Answer:0.3

Step-by-step explanation:

2 divided by 0.75 full divison work i dont just need the answer​

Answers

Answer:

0.375

Step-by-step explanation:

Check the picture below.

whenever we do division of decimals, we have to mind how many decimals are there on each amount, the dividend as well as the divisor, that way we pad with zeros the other amount accordingly whilst losing the dot, for example, to say divide 3 by 0.123, 3 has no decimals, whilst 0.123 has three decimals, so we can just divide 3000 by 0123, so dividing 3 by 0.123 is the same as dividing 3000 by 123.  Another example, if we were to divide say 23.761 by 555.89331, the dividend has 3 decimals, that means 3 zeros the other way, the divisor has 5 decimals, that means 5 zeros the other way while losing the dots, so we'd end up dividing 2376100000 by 55589331000, which we can simplify to just 2376100 by 5589331, as you can see in the picture in this case.

11x+7y=17
solve for y

Answers

[tex]\implies {\blue {\boxed {\boxed {\purple {\sf {\: y = \frac{17 - 11x}{7} }}}}}}[/tex]

[tex]\large\mathfrak{{\pmb{\underline{\red{Step-by-step\:explanation}}{\red{:}}}}}[/tex]

[tex]\\11x + 7y = 17[/tex]

[tex] \\➺ \: 7y = 17 - 11x[/tex]

[tex]\\➺ \: y = \frac{17 - 11x}{7} [/tex]

[tex]\bold{ \green{ \star{ \orange{Mystique35}}}}⋆[/tex]

Simplify:......................................................

Answers

Answer:

...

Step-by-step explanation:...

The cut off number is 2x-1

A person walks 1/6 mile in 1/18 hour.

The person's speed is _ miles per hour.

Answers

This Is What I Got!

Hope This Helps! :)

Have A Good Day!!

And If You Can I Wouldn't Mind A Brainliest! :))

Answer:

Divide 1/6 miles to 1/12hour since u wanna find our miles per hour

So it’ll be : 1/6 / 1/12

= 1/6 x 12/1

= 2 miles

On a coordinate plane, a line goes through (negative 3, negative 3) and (negative 1, 5). What is the equation of the line parallel to the given line with an x-intercept of 4?

Answers

y = mx + c

m = gradient

gradient of line:
[5 - (-3)]/[(-1) - (-3)]
= 8/2
= 4

y = mx + c
subsitute (4, 0)
0 = (4)(4) + c
0 = 16 + c
c = -16

equation of the line:

y = 4x - 16

hope this helped :)



Answer:

4, -16

Step-by-step explanation:

i need to see the steps for simplifying 3(m-5)+m​

Answers

Answer:

4m - 15

Step-by-step explanation:

a( x + y) = ax + ay

[tex]3( m - 5 ) + m\\\\3m - 15 + m \\\\4m - 15[/tex]

Answer:

4m-15

Step-by-step explanation:

Distrubite 3 through the parentheisis

3m-15+m

Collect like terms

4m-15

What is the value of the expression 3m-4.2 If m equals 2.1

Answers

Answer:

2.1

Step-by-step explanation:

Given :

3m-4.2 where, m=2.1

Now,

3(2.1)-4.2

6.3-4.2

2.1

Answer is 2.1

Calculate the area of the following Circle

Answers

Answer:

324

Step-by-step explanation:

Circles are 100% even. If you add another line going opposite direction you would multiply 18x18. You get 324.

Stats question what are the main difference between frequenting and Bayesian

Answers

Answer:

The frequentist believes that probability represents long term frequencies of repeatable events (such a flipping a coin). Frequentists do not attach probabilities to hypotheses or unknown values. On the other hand, the Bayesian approach uses probabilities to represent the uncertainty in any event or hypothesis.

HOPE IT HELPS

What is the average score of runa 140,96 and 13?​

Answers

[tex]\huge\textsf{Hey there!}[/tex]

[tex]\large\textsf{Formula:}[/tex]

[tex]\mathsf{\dfrac{Sum\ of\ all\ terms}{Total\ number\ of\ values\ in\ the\ data}= your\ mean/average}[/tex]

[tex]\mathsf{\dfrac{140+96+13}{3}}[/tex]

[tex]\mathsf{140+36+13}[/tex]

[tex]\mathsf{140 + 96 = \bf 236}[/tex]

[tex]\mathsf{236 + 13}[/tex]

[tex]\mathsf{ = \bf 249}[/tex]

[tex]\mathsf{\dfrac{249}{3}}[/tex]

[tex]\mathsf{= \bf 83}[/tex]

[tex]\boxed{\boxed{\large\textsf{Possible answer: \huge \bf 83}}}\huge\checkmark[/tex]

[tex]\large\textsf{Good luck on your assignment and enjoy your your day!}[/tex]

~[tex]\frak{Amphitrite1040:)}[/tex]

Suppose that g(x)= f(x)+ 6. Which statement best compares the graph of g(x) with the graph of f(x)?

A. The graph of g(x) is the graph of f(x) shifted 6 units down.

B. The graph of g(x) is the graph of f(x) shifted to the right.

C. The graph of g(x) is the graph of f(x) shifted 6 units to the left.

D. The graph of g(x) is the graph of f(x) shifted 6 units up.

Answers

Answer:

D

Step-by-step explanation:

The + 6 moves it up 6 units.

The correct answer is (D) "The graph of g(x) is the graph of f(x) shifted 6 units up."

What is the function?

A relationship between a group of inputs and one output is referred to as a function. In plain English, a function is an association between inputs in which each input is connected to precisely one output. A domain, codomain, or range exists for every function. Typically, f(x), where x is the input, is used to represent a function.

When we add a constant to a function, such as in the case of g(x) = f(x) + 6, it will shift the graph of f(x) upward by 6 units.

This is because, for any value of x, the value of f(x) will be added to 6, resulting in a vertical shift of the entire graph.

Option (A) is incorrect because adding 6 to f(x) would shift the graph up, not down.

Option (B) is incorrect because adding a constant to a function does not cause it to shift horizontally.

Option (C) is incorrect because adding 6 to f(x) would shift the graph right, not left.

D. The graph of g(x) is the graph of f(x) shifted 6 units up. Adding a constant term to a function will shift the graph of the function vertically. In this case, adding 6 to f(x) will shift the graph of f(x) upward by 6 units, resulting in the graph of g(x).

Learn more about function here:

https://brainly.com/question/29633660

#SPJ7

Given points (-3;-6), G(3; -2) and H(6; 1); determine:
(a) The equation of line FG

Answers

Answer:

The equation of line FG is [tex]y = \frac{2}{3}x - 4[/tex]

Step-by-step explanation:

Equation of a line:

The equation of a line has the following format:

[tex]y = mx + b[/tex]

In which m is the slope and b is the y-intercept.

F(-3;-6), G(3; -2)

When we have two points, the slope is given by the change in y divided by the change in x. So

Change in y : -2 - (-6) = -2 + 6 = 4

Change in x: 3 - (-3) = 3 + 3 = 6

Slope: [tex]m = \frac{4}{6} = \frac{2}{3}[/tex]

So

[tex]y = \frac{2}{3}x + b[/tex]

Finding b:

(3; -2) means that when [tex]x = 3, y = -2[/tex]. We use this to find b.

[tex]y = \frac{2}{3}x + b[/tex]

[tex]-2 = \frac{2}{3}(3) + b[/tex]

[tex]2 + b = -2[/tex]

[tex]b = -4[/tex]

The equation of line FG is [tex]y = \frac{2}{3}x - 4[/tex]

Solve:
n = 8
4
O n = 2
0 n = 12
Ô n = 24
0 n = 32

Answers

Answer:

1/4 n = 8

n = 8 * 4

n = 32

so option 4 i.e.

n = 32. is the ans

Answer:

n=32

Step-by-step explanation:

Given :

[tex]\frac{1}{4} n=8[/tex]

Now,

Value of n can be calculated as :

[tex]\frac{1}{4} n=8[/tex]

n=8×4

n=32

Therefore, option (D) is correct.

Can someone answer with steps and explanation? Thanks.

Answers

Answer:

[tex]x=-16\text{ or } x=7[/tex]

Step-by-step explanation:

Since ΔABC is mapped onto ΔDEF, we can write that:

[tex]\Delta ABC\cong \Delta DE F[/tex]

By CPCTC:

[tex]\angle A\cong \angle D[/tex]

And since ΔABC is isosceles with Vertex C:

[tex]\angle A \cong \angle B[/tex]

We are given that:

[tex]m\angle D=34[/tex]

Hence:

[tex]m\angle A=34=m\angle B[/tex]

We are also given that:

[tex]m\angle C=x^2+9x[/tex]

The interior angles of a triangle must sum to 180°. Thus:

[tex]m\angle A+m\angle B+m\angle C=180[/tex]

Substitute:

[tex](34)+(34)+(x^2+9x)=180[/tex]

Simplify:

[tex]68+x^2+9x=180[/tex]

Isolate the equation:

[tex]x^2+9x-112=0[/tex]

Factor:

[tex](x+16)(x-7)=0[/tex]

Zero Product Property:

[tex]x+16=0\text{ or } x-7=0[/tex]

Solve for each case:

[tex]x=-16\text{ or } x=7[/tex]

Testing the solutions, we can see that both yields C = 112°.

Hence, our solutions are:

[tex]x=-16\text{ or } x=7[/tex]

international system of 89643092 in words​

Answers

Answer:

Eighty nine million six hundred forty three thousand ninety two

Step-by-step explanation:

89,643,092

=> Eighty nine million six hundred forty three thousand ninety two

he speeds (in MPH) of automobiles traveling in a city are given below:
20, 35, 42, 52, 65, 49, 24, 37, 23, 41, 50, 58
The mean speed of the cars is

Answers

Answer:

Mean speed = 41.3 mph

Step-by-step explanation:

Given that,

The speeds of an automobiles are given below:

20, 35, 42, 52, 65, 49, 24, 37, 23, 41, 50, 58

We need to find the mean speed of the cars.

Mean = sum of observations/ no. of observation

[tex]M=\dfrac{20+35+42+52+65+49+24+37+23+41+50+58}{12}\\\\M=41.3[/tex]

So, the mean speed of the cars is equal to 41.3 mph.

seventy four lakh nine thousand eleven in numeral form​

Answers

Seventy Four Lakh , Nine Thousand And Eleven=

7409011

I need help k please help gardinuhola

Answers

Answer:

Step-by-step explanation:

for circle

diameter = 26 cm

radius = diamtere/2

=26/2

=13 cm

area of circle  = πr^2

=3.14^13^2

=3.14*169

=530.66 cm^2

=531 cm^2   (after round off )

area of square= l^2

=5.1^2

=26.01 mi^2

are of rectangle = l *b

=6*5.1

=30.6 m^2

area of triangle = base*height / 2

=9*6.4 / 2

=57.6 / 2

=28.8 yd^2

4(x+9)=8x-7
Solve for x

Answers

Answer:

x = 10.75

Step-by-step explanation:

4(x+9)=8x-7

Step 1: Distribute the 4 to the x and 9

4 * x = 4x

4 * 9 = 36

We now have 4x + 36 = 8x - 7

Step 2: add 7 to both sides

36 + 7 = 43

-7 + 7 cancels out

We now have 4x + 43 = 8x

Step 3 subtract 4x from both sides

4x - 4x cancels out

8x - 4x = 4x

We now have 4x = 43

step 4 divide both sides by 4

4x / 4 ( the 4s cancel out and we're left with x )

43/4 = 10.75

We're left with x = 10.75

Find the area of the composite figure.

Answers

Answer:

25m²

Step-by-step explanation:

it might be helpful for you

Determine whether the following event is mutually exclusive or not mutually exclusive.

Choosing a student who is a mathematics major or a business major from a nearby university to participate in a research study. (Assume that each student only has one major.)

Answers

Answer:

The event is mutually exclusive.

Step-by-step explanation:

Mutually exclusive events are events that cannot exist simultaneously.

Thus, events that are not mutually exclusive can exist simultaneously.

Since each student only has one major, a single student cannot be both a mathematics major and a business major.

So, the event is mutually exclusive.

xác định m để 3 vector sau đây phụ thuộc tuyến tính
u=(m+1,1,m+1),v=(1,1,1),u(2,0,m+2



0

Answers

Answer:

i dont know hahaha

Step-by-step explanation:

soryy

What is the area of this figure?

Answers

Answer:

90km² only if it is parallelogram

Step-by-step explanation:

base = 9km

height=10km

area of parallelogram = b x h

=9km x 10km

=90km²

Answer:

A = 90km2

Step-by-step explanation:

Area of a rhombus is:

1. A = s x h (if given side and height)

2. A = 1/2 a x b (if given lengths of diagonals)

3. A = s^2 sin A (if given side and length)

Therefore from your problem, height and side is given thus, you'll use number 1

A = s x h

A = 9km x 10km = 90km2

How many liters each of a 25% acid solution and a 50% acid solution must be used to produce 80 liters of a 40% acid solution?

Answers

Answer:

32 and 48 liters

Step-by-step explanation:

Let 25% solution is x liters, then 50% solution is (80 - x) liters.

Acid content is going to be same:

0.25x + 0.5(80 - x) = 80*0.40.25x - 0.5x + 40 = 320.25x = 8x = 8/0.25x = 32 liters

So 32 liters of 25% solution and 80 - 32 = 48 liters of 50% solution

Is 9/54 equivalent to 1/3

Answers

Answer: No.

Step-by-step explanation:

9/54 is equal to 1/6

Answer:

no, it is not

Step-by-step explanation:

9/54 can be simplified by dividing both sides by 3, getting 3/18. you can do this again and get 1/6. significantly smaller that l/3

Can you solve this problem

Answers

Answer:

x = 18

Step-by-step explanation:

8 x 18 - 3 = 141

The answer is to your question is 18
Other Questions
Select the correct answer.As society has changed, what have most households become?O A.dual incomeB. single incomeOC.controlled by womenD.compromised of extended family caretakers At Dubai English School, 549 students use buses to go to school. If this number is 75% of the total school enrollment, then how many students are enrolled in total? O O OFrom what you have read so far, which of the following describe a "deeper message" of The Awakening?Check the two best choices.Creole families spend the summer on an island.Edna interacts with her husband and other men in different ways.Women should question their identities and roles in society.Differences in the way women treat their children reveal character traits.It's difficult to go against society's expectations.DO NEVIntro Which is the Value if this expression when m = 3 and n = -5 A box with a square base and open top must have a volume of 256000 c m 3 . We wish to find the dimensions of the box that minimize the amount of material used. First, find a formula for the surface area of the box in terms of only x , the length of one side of the square base. In an oxidation-reduction reaction, the substance oxidized always ________. a. takes on oxygen atoms b. gives up hydrogen atoms c. gains electrons d. loses electrons e. becomes a charged species Neil is buying steak for a cookout on Saturday. Steak is on sale for $9.62 per pound. If he buys 7.5 pounds of steak, how much money does he spend? A payday loan company charges a $90 fee for a $500 payday loan that will be repaid in 16 days.Treating the fee as interest paid, what is the equivalent annual interest rate? the assignment for this lesson is to compare and contrast the objectivity of two sources,representing two different media,that discuss the same topic or event. you must also provide an objective summary of the sources to show the reader thar you have a goood understanding of the content IlusWhat are the gross sales?Income StatementFor the Year Ended August 31, 2012RevenuesStudent Lessons$29,520.00Recital Sales15,900.00Total Revenues:45,420.00ExpensesCostumes and AccessoriesBuilding MaintenanceTotal Expenses:16,700.0011,320.0028,020.00*Call numbers in 000s) a phlebotomy technician has collected a neonatal screening card. which of the following actions should the technician take to properly dry the blood stop collection card? Why do the poles experience about six months day and six months night? critically discuss three nagative reosons why some people join protest action The Jenkins family's monthly budget isshown in the circle graph. The family has amonthly income of $4,800. How muchmoney do they spend on transportationeach month?Emergency fund5%Transportation5%Housing30%Medical22%Food15%Clothing6%Entertainment7%Savings10% Write an essay expressing your own thoughts as a student now in the midst of a pandemic.It only takes 7-10 sentencesNonsense =report give your answer in detail plzz If you were a teacher , how would you make you online class better? Explain the significance and impact of the American with Disabilities Act on businesses in the United States. Scientists have been studying the size of three populations in a river in Ireland. In this ecosystem, pike (a type of big fish), and perch eat snails. The data showed that all three populations were stable. Then the snail population increased suddenly. What will likely happen to the size of the perch population as a result? The size of the perch population will...Options:A. Stay the same. The size of it's consumer population did not change, so the number of deaths in the perch population did not change. There will be the same number of births and deaths in the perch population.B. Decrease. The larger snail population will take more energy storage molecules from the ecosystem, leaving fewer available for the perch population. This will lead to more deaths than births in the perch population.C. Increase. The larger snail population will provide more energy storage molecules for the perch population, so they will reproduce more. This will lead to more births than deaths in the perch population.D. Increase. An increase in any population within an ecosystem will lead to an increase in the sizes of all other populations in the ecosystem. TASKIEND OF THE YEAR - ESCAPE ROOMMovie ThemeFnd the area and circumference of each figure below. Use 31 for it. If needed,round your answers to the nearest hundredths placeCODE Type the numeral answer. NO LABELS Part a crea, circumference, part barea, circumference For example 765, 326, 2004, 3563610 mmMOVIETICKETYour answer